Computer operating system software has become increasingly standardized. But when a large business with multiple, lin...

Julie-V on August 27, 2019

Answer Explanation

Hi LSAT Max, Would "unauthorized access to all the computers at the same time could be virtually eliminated" from the stimulus support the part of the answer choice that mentions "but will not have protected every computer from viral invasion"? Thanks!

Reply
Create a free account to read and take part in forum discussions.

Already have an account? log in

laurenkirwan on December 22, 2019

Please reply @LSATmax